0% found this document useful (0 votes)
19 views17 pages

Physics Qualifier Round - Problems & Solutions - STEMS 2021

The document contains hints and solutions for the STEMS 2021 Physics Qualifier Round, including problems on center of mass, lens magnification, resistance in wires, cone of illumination, Earth's orbital eccentricity, sound frequency in water, and random walks. Each problem is followed by a detailed solution that includes relevant formulas and calculations. The solutions cover various physics concepts and provide numerical answers where applicable.

Uploaded by

sher
Copyright
© © All Rights Reserved
We take content rights seriously. If you suspect this is your content, claim it here.
Available Formats
Download as PDF, TXT or read online on Scribd
0% found this document useful (0 votes)
19 views17 pages

Physics Qualifier Round - Problems & Solutions - STEMS 2021

The document contains hints and solutions for the STEMS 2021 Physics Qualifier Round, including problems on center of mass, lens magnification, resistance in wires, cone of illumination, Earth's orbital eccentricity, sound frequency in water, and random walks. Each problem is followed by a detailed solution that includes relevant formulas and calculations. The solutions cover various physics concepts and provide numerical answers where applicable.

Uploaded by

sher
Copyright
© © All Rights Reserved
We take content rights seriously. If you suspect this is your content, claim it here.
Available Formats
Download as PDF, TXT or read online on Scribd
You are on page 1/ 17

Hints and Answers

STEMS 2021
Physics Category A
Qualifier Round
December 31, 2020

1. Consider the XY plane with n point particles of equal mass m


distributed on the unit circle in the form of a regular n−gon. Now,
one of the masses is removed and its mass is equally redistributed
among the other particles. If Sn denotes the shift in the centre of
mass, find the value of

!
exp 1 + ∑ (−1)m Sm
m =3
b

b b
Solution. After redistribution each mass becomes,
m n
m0 = m + = m
n−1 n−1 b
A

If we exclude the mass A, the centre of mass of the remaining


n − 2 masses is at the centre of the circle. b b

If we now include A, the centre of mass would be at,


b
m0 ×r 1 Figure 1: For n = 8 after redistribution.
Sn = =
( n − 1) m 0 n−1

since we have a unit circle.


So,

1 1
1+ ∑ (−1)m Sm = 1 − 2 + 3 − . . .
n =3

which is an alternating harmonic series 1 and it sums to ln(2). 1


Search for “1 − 1/2 + 1/3 series” on
Google to get a direct answer.
Therefore,
exp(ln(2)) = 2
is the answer2 . 2
The exp and the ln are inverses of each
other. What we mean is, exp(ln( x )) = x
2. Consider the following arrangement of spherical lenses. Each lens and ln(exp( x )) = x.

has a focal length f and the positions of the lenses are as given in
the diagram below.
If the object is placed upright at a distance f /2 on the left of the
first lens, find the net magnification produced by the combination
of three lenses.
hints and answers stems 2021 physics category a qualifier round 2

• •

f /2 f f

Solution. There are two ways to solve this problem. One is by ray
tracing. The other is by the lens formula,
1 1 1
+ =
u v f
For the first lens,
2 1 1
+ = =⇒ v = − f
f v f
and the magnification is,
v
m1 = − =2
u
Now this image is at a distance 2 f to the left of the second lens.
So,
1 1 1
+ = =⇒ v = 2 f
2f v f
and the corresponding magnification is,
v
m2 = − = −1
u
The image due to the second lens is therefore at a distance of f to
the right of the third lens. So,
−1 1 1 f
+ = =⇒ v =
f v f 2
and the corresponding magnification is,
v 1
m3 = − =
u 2
The net magnification is a product of the above three magnifica-
tions and therefore,

m = m1 m2 m3 = −1

is the answer.
hints and answers stems 2021 physics category a qualifier round 3

3. Consider a wire of length L and area of cross-section A. We cut


up the wire into 4 equal parts (we mean equal lengths) to get
wires W1 ,W2 ,W3 and W4 . Now, each wire Wi (where i = 1, 2, 3, 4)
is stretched to i times its length. Then we put the larger three
resistances in parallel and combine this in series with the smallest
resistance. If the effective resistance is Re = aR/b, where R is the
resistance of the original wire and a, b are positive integers with
( a, b) = 1, then a − b is:

Solution. Suppose the resistivity of the wire is ρ. The idea is that


when we stretch a wire, it’s volume doesn’t change. The lengths
of the wires after stretching are, L/4, L/2, 3L/4, L and the cross-
sectional areas are A, A/2, A/3, A/4 respectively.
Then we apply the formula,

ρL
R=
A
and get the resistances to be,

ρL ρL 9ρL 4ρL
R1 = , R2 = , R3 = , R4 =
4A A 4A A
If we put the larger three resitances (R2 , R3 , R4 ) in parallel, the
resultant resistance is,
1 1 1 1 36 ρL
= + + =⇒ R p =
Rp R1 R2 R3 61 A

Combining this in series with R1 , the resultant resistance is,

205 ρL 205
R 0 = R1 + R p = = R
244 A 244
where R is the resistance of the original wire. Hence,

a − b = −39

is the answer.

4. There is a thin layer of oil of thickness h on top of a lake. Consider


a point at the bottom of the lake. If the cone of illumination of that
point inside water has volume V = k( H − h)3 , find the value of k
upto two decimal places, H is the depth of the lake. Take π = 3.14,
refractive index of water = 1.3, refractive index of oil = 1.518.

Solution. The radius of the cone is, β

r = ( H − h) tan(α)

where α is as shown in the figure.


α

Figure 2: The path light takes


hints and answers stems 2021 physics category a qualifier round 4

The volume of the cone is therefore,


1 2 1
V= πr ( H − h) = π tan2 (α)( H − h)3
3 3

From Snells’ law at the water-oil boundary,

ηwater sin(α) = ηoil sin( β)

at the oil-air boundary,

ηoil sin( β) = ηair sin(90◦ ) = 1

So we get,
1
sin(α) =
ηwater
Now,
1
tan(α) = q
2
ηwater −1
Substituting this in the volume we finally get:

1 1
V= π 2 ( H − h )3
3 ηwater − 1

Substituting the values given in the question we get,

k ≈ 1.52

rounded off to two decimal places.

5. We will employ a crude model to estimate the eccentricity of the


Earth’s orbit around the sun. We will suppose the intensity of
Sun’s radiation falls as inverse distance squared and say the tem-
perature we measure at the equator is proportional to this inten-
sity.
Say the highest temperature during summer at the equator is
about 50◦ C and the lowest temperature at the equator during
winter is 7◦ C.
Using this information, estimate the eccentricity of the Earth’s
orbit around the sun. Neglect the radius of the Earth which is very
small compared to the distance between the Earth and Sun. The
following conversion formula may be helpful, 0◦ C = 273.16 K.
Express your answer upto 3 decimal places.

Solution. The temperature is the lowest when the Earth is farthest


a (1 + e ) a (1 − e )
from the sun and highest when it is closest.
The earth moves in an elliptic path with the sun at the focus.
Figure 3: The orbit of the Earth around
the sun. The lowest temperature (in
Winter) corresponds to the distance
a(1 + e) and the highest temperature (in
Summer) corresponds to a(1 − e).
hints and answers stems 2021 physics category a qualifier round 5

The model says that the temperature is inversely proportional to


the distance squared,
1
T∼ 2
r
Therefore,
Tmin (1 − e )2
=
Tmax (1 + e )2
Solving for the eccentricity we get,
√ √
Tmax − Tmin
e= √ √ ≈ 0.036
Tmax + Tmin

6. Consider a cylindrical column of water kept just below the boiling


point at 100◦ C. We bring in a tuning fork of unknown frequency f
to the open end of the column and observe strong ripples on the
surface of the water. Now we slowly add heat and the water starts
evaporating. After supplying 4.77 × 107 J of heat to the water, we
see strong ripples again. Determine the unknown frequency f in
Hz (express your answer to the nearest integer).
This whole experiment is being performed at room temperature
(∼ 30◦ C) and suppose that all the heat supplied goes into evapo-
rating the water. Some useful numbers are: the radius of the cylin-
drical column is r = 3 cm, density of water at 100◦ C is ρ = 9586.5
kg/m3 , speed of sound in air at 30◦ C is c = 343 m/s, latent heat of
vaporization of water is, L = 2257 × 103 J/kg.

Solution. The water vessel acts like a resonating column with one
closed end and one open end. We must therefore have a node at λ
4
the surface of water (closed end) and an anti-node at the mouth of
the vessel (open end).
Water
When heat is supplied, the water level decreases in the vessel. This
decrease must be such that there is resonance again.
Therefore the required decrease in the water level must be λ/2 Figure 4: Before evaporation. Figure not
to scale.
where λ is the wavelength of the sound produced by the tuning
fork.
The mass of water evaporated is therefore,
λ
m = ρ πr2
2 3λ
4
where r is the radius of the cylindrical vessel and ρ is the density
of water.
The amount of heat to be supplied is therefore, Water

λ c Figure 5: After evaporation. Just a


∆Q = Lm = Lρ πr2 = Lρ πr2 rough sketch, not to scale.
2 2ν
hints and answers stems 2021 physics category a qualifier round 6

where c is the speed of sound and ν is the frequency of the tuning


fork. Solving for ν, we obtain:
ν ≈ 220 Hz
rounded off to the nearest integer.

7. A drunkard performs a random walk on an infinite straight line


after a party (each step the drunkard takes is either forward or
backward). Due to increasing fatigue, each step taken is 0.99 times
the previous one. If the initial step was 1 unit, the RMS distance
from the starting point after N steps is L N . Compute,
lim L N .
N →∞
(Express your answer to one decimal place)

Solution. The position of the drunkard after N steps is:


x N +1 = a0 + (0.99) a1 + (0.99)2 a2 + . . . (0.99) N a N
where a0 , a1 , . . . , a N take values ±1 depending on the drunkard’s
choice.
For example if the drunkard’s first four steps are forward and the
next step is backward, then his position would be:
x6 = 1 + 0.99 + 0.992 + 0.993 + 0.994 − 0.995

Now since the drunkard has equal chance of going either forward
or backward at every step3 , the average value of each ai is 0. 3
This is what we mean by random walk

h ai i = 0
for i = 1, 2, 3, . . . .
Similarly4 , 4
ai a j can take values either 1 or −1.
h ai a j i = 0 If ai a j = 1, then both ai and a j are
1 or both are −1. The probability of
when i 6= j and i, j = 1, 2, 3, . . . . this happening is 1/2. Similarly the
probability that ai a j is −1 is also 1/2.
When i = j, we have a2i which is always 1. So,
h a2i i = 1
for all i = 1, 2, 3, . . . .
Squaring the expression for x N +1 and taking the average we there-
fore get,
L2N = h x2N +1 i = 1 + (0.99)2 + (0.99)4 + · · · + (0.99)2N
In the limit N → ∞ we get:
s
1
lim L N = ≈ 7.1
N →∞ 1 − (0.99)2
Hints and Answers
STEMS 2021
Physics Category B
Qualifier Round
December 31, 2020

1. The figure below represents a section of a capacitor whose square


plates (shown with a ruled line) AB and CD are inclined at an
angle of 30◦ . The plates have a length of 1 m. Suppose the electric
field lines between the plates are circular arcs centered around O
and we ignore the edge effects. The medium between the capacitor
plates is air whose dielectric constant is taken to be k = 1. The
capacitance of the system is a × 10−12 F. The nearest integer to a is:

1m

B
1m
A
O 30◦

1m
C
1m D

Solution. Suppose the potential difference across the capacitor


plates is V. Consider an infinitesimal length dr of the capacitor at a
distance r from O. The separation between the plates at this radius
is rφ where φ = π/6. The electric field at the surface of the plates
is therefore1 , V/rφ. 1
Since the electric field lines were
assumed to be circular with center at O
We also know that the electric field near the surface of a conduct-
ing plate is σ/e0 where σ is the charge denisty. Hence we get,

Ve0
σ (r ) =

The total charge is therefore2 , 2


From the figure in the question it is
clear that r varies from 1 to 2.
Z 2 Z 2
Ve0 1 Ve0
Q= σ (r )dr = dr = ln(2)
1 φ 1 r φ

The capacitance is therefore,

Q e
C= = 0 ln(2)
V φ
hints and answers stems 2021 physics category b qualifier round 2

If we now substitute the numbers we get:

C ≈ 1.172 × 10−11 ≈ 12 × 10−12

and so,
a = 12

2. Suppose two particles A and B perform Simple Harmonic Motion


with the same amplitude and frequency ω, but B is ahead of A in
phase by π/4.
Starting from the point where A is at the origin, the time after

which they meet for the third time is ; where gcd(m, n) = 1.

Then m + n is:

Solution. Since A is at the origin when t = 0,

x A (t) = A sin(ωt)

B is π/4 ahead of A with the same amplitude and so,

x B (t) = A sin(ωt + π/4)

The simplest way to solve the problem is to plot the two functions.
This would therefore give us,
19π
t=

and hence,
m + n = 27
Figure 1: The two graphs meet for the
3. The entropy of a Schwarzschild Black Hole is, S = M2 /2, where third time at ωt = 19π/8.
M is the mass. The internal energy of a black hole is taken to be
it’s mass energy U = Mc2 and we work in units where c = 1.
Suppose the amount of heat required to increase the temperature
of the black hole from a given value T to ∞ is aT b . What is a − b?

Solution. The mass M plays the role of internal energy and we


have the relation, S = M2 /2. In particular, the temperature of the
black hole is:
1 dS 1
= =⇒ T =
T dM M
We may then write S = 1/2T 2 . The amount of heat required for
the process is therefore3 , 3
The negative sign is counter-
Z ∞ intuitive. This is related to the fact
1 1
Z Z
that Schwarzschild Black Holes have
dQ = TdS = − dT = −
T T2 T a negative specific heat. The black
hole emits heat while increasing it’s
Therefore, a = b = −1 and correspondingly, temperature.

a−b = 0
hints and answers stems 2021 physics category b qualifier round 3

4. The goal of this problem is to obtain a relationship between the


rms speed of stars in a galaxy, the galaxy’s mass and the galaxy’s
radius. You will be using the Virial Theorem which states,
1
h T i = − hV i
2
where h T i is the average kinetic energy of the system and hV i is
the average potential energy of a mechanical system.
For simplicity, we consider the galaxy is a spherical mass distri-
bution with total mass M, radius R and average density ρ. It’s
potential energy may therefore be determined and we know V in
the above expression.
To determine h T i we suppose the galaxy is made up of N stars
each of the same mass m. Then the total kinetic energy is:
N
1
T= ∑ 2 mv2i
i =1

Since there is nothing special about any one particular star, by


a symmetry argument we may say that hv2i i is the same for all
i = 1, . . . , N. Now we write,

hv2 i = hv2x i + hv2y i + hv2z i

and since there is nothing special about any one particular direc-
tion, each term on the RHS may be assumed equal to a quantity
we denote by σ.
Then we have,
a GM
σ=
b R
What is a + b?

Solution. The potential energy of a unifrom spherical mass distri-


bution of mass M and radius R is4 , 4
For a derivation refer to HC Verma’s
Concepts of Physics.
3 GM2
V=−
5 R

The average kinetic energy is,


N
3 3
hT i = ∑ 2 mσ = 2 Mσ
i =1

By the Virial theorem,


1 GM
σ=
5 R
and so5 , 5
The point of this problem was to give
a+b = 6 the candidate a sense of the kind of
crude approximations that are generally
used in physics. For more interesting
applications of the Virial Theorem, refer
to Ladera, C. L., Alomá, E., & León,
P. (2010). The Virial Theorem and its
applications in the teaching of Modern
Physics. Latin-American Journal of
Physics Education, 4(2), 1. which may
be found at: http://www.lajpe.org/
may10/01_Celso_Ladera.pdf
hints and answers stems 2021 physics category b qualifier round 4

5. At a given pressure P, the transition from one phase of matter


to another proceeds at a constant fixed temperature T ( P). This
transition is called a first-order phase transition. For example, ice
melts at 0◦ C at atmospheric pressure and this temperature remains
constant until all the ice turns into water.
When the external pressure changes, this constant temperature
changes too and the differential equation governing this change is:

dP L
=
dT T ( v2 − v1 )

where L is the latent heat of this transition, T is the constant tem-


perature at which the transition occurs and v1 , v2 are the specific
volumes (inverse of density) of the initial and final phases respec-
tively.
Suppose the latent heat and the densities of the two phases are
constant, then solve the above equation and obtain P( T ). Using
this relation, estimate at what pressure (in atm) does ice melt at a
temperature T = −1◦ C (express you answer to the nearest integer).
The relevant values are: ρwater = 1000 kg/m3 (at 0◦ C), ρice = 920
kg/m3 (at 0◦ C), the latent heat of fusion for ice is, L = 3.34 × 105
J/kg and atmospheric pressure is 1 atm = 101325 Pa.

Solution. Solving the differential equation we get:


 
L T
P( T ) = P0 + ln
v2 − v1 T0

We know that ice melts at T0 = 273.16 K at atmospheric pressure


P0 = 1 atm.
The process is ice melting to water. So, v1 = 1/ρice and v2 =
1/ρwater .
When T = 272.16 K, we therefore have:

3.34 × 105
 
272.16
P = 101325 + 1 1
ln
1000 − 920
273.16

Converting Pascal to atm, we get:

P ≈ 140

6. We will employ a crude model to estimate the eccentricity of the


Earth’s orbit around the sun. We will suppose the intensity of
Sun’s radiation falls as inverse distance squared and say the tem-
perature we measure at the equator is proportional to this inten-
sity.
hints and answers stems 2021 physics category b qualifier round 5

Say the highest temperature during summer at the equator is


about 50◦ C and the lowest temperature at the equator during
winter is 7◦ C.
Using this information, estimate the eccentricity of the Earth’s
orbit around the sun. Neglect the radius of the Earth which is very
small compared to the distance between the Earth and Sun.
Express your answer upto 3 decimal places.
The following conversion may be helpful, 0◦ C = 273.16 K.

Solution. The temperature is the lowest when the Earth is farthest


a (1 + e ) a (1 − e )
from the sun and highest when it is closest.
The earth moves in an elliptic path with the sun at the focus.
Figure 2: The orbit of the Earth around
The model says that the temperature is inversely proportional to the sun. The lowest temperature (in
the distance squared, Winter) corresponds to the distance
1 a(1 + e) and the highest temperature (in
T∼ 2 Summer) corresponds to a(1 − e).
r
Therefore,
Tmin (1 − e )2
=
Tmax (1 + e )2
Solving for the eccentricity we get,
√ √
Tmax − Tmin
e= √ √ ≈ 0.036
Tmax + Tmin

7. A drunkard person performs a random walk on an infinite plane


after a party. Due to increasing fatigue, each step taken is 0.99
times the previous one. If the initial step was 1 unit, the RMS
distance from the starting point after N steps is L N . Compute,

lim L N
N →∞

(Express your answer to one decimal place)

Solution. We will take the Argand plane and use complex num-
bers instead of R2 .
The position of the drunkard in the argand plane after n steps is:

zn+1 = exp(iθ0 ) + 0.99 exp(iθ1 ) + · · · + (0.99)n exp(iθn )

where θ0 , θ1 , . . . , θn are the directions of the first, second, . . . , nth


step respectively. Figure 3: A typical path of the drunk-
ard (figure not to scale)
We write this as,
n
z n +1 = ∑ (0.99)k exp(iθk )
k =0
hints and answers stems 2021 physics category b qualifier round 6

Now,
n
|zn+1 |2 = z∗n+1 zn+1 = ∑ (0.99)k+ j exp(i(θk − θ j ))
k,j=0

Say Ln is the RMS value of the position of the drunkard, then:


n n
L2n = h|zn+1 |2 i = ∑ (0.99)k+ j hexp(i(θk − θ j ))i = ∑ (0.99)2k
k,j=0 k =0

where we used the fact that6 , hexp(i (θk − θ j ))i = δjk when θk and 6
The Kronecker-Delta is defined as,
θ j are random.
(
1, j = k
δjk =
If we take the limit n → ∞, we get: 0, j 6= k

s
1
lim Ln = ≈ 7.1
n→∞ 1 − (0.99)2

rounded off to one decimal place.


Hints and Answers
STEMS 2021
Physics Category C
Qualifier Round
December 31, 2020

1. Suppose a particle moves in a plane under the influence of a cen-


tral force of the form:

V (r ) = Ar −q + Br − p

Suppose the trajectory of the particle is a spiral of the form r (θ ) =


Cθ 4 , with c constant. What is the value of p + q? (Upto one digit
after the decimal).

Solution. The idea is to obtain an equation relating r (θ ) and V (r )


and compare the powers of θ on both sides. The Lagrangian for a
central potential is,
1
L= m(ṙ2 + r2 θ̇ 2 ) − V (r )
2
and the equations of motion are,

mr2 θ̇ = l
dV (r )
mr̈ = − + mr θ̇ 2
dr
where l is an integration constant (angular momentum). Using
the first equation eliminate the time derivatives in the second
equation. After some hard work1 , we get: 1
Alternatively one could follow the
approach given in Goldstein by intro-
2
2l 2 l 2 d2 r l2 ducing u = 1/r. For more details, see

dr dV
− + =− + 3 Page 86 (Equation 3-34b) in Classical
mr5 dθ 4
mr dθ 2 dr mr Mechaincs, Herbert Goldstein, 2nd
edition, Addison-Wesley Publishing
Substitute, r (θ ) = Cθ 4 and V (r ) = Ar − p + Br −q and compare the Company (1980).
co-efficients of θ, we’d get:

−4( p + 1) = −14
−4(q + 1) = −12

and so, p = 2.5, q = 2 and the answer is p + q = 4.5 .

2. The entropy of a Schwarzschild Black Hole is, S = M2 /2, where


M is the mass. The internal energy of a black hole is taken to be
it’s mass energy U = Mc2 and we work in units where c = 1.
Suppose the amount of heat required to increase the temperature
of the black hole from a given value T to ∞ is aT b . What is a − b?
hints and answers stems 2021 physics category c qualifier round 2

Solution. The mass M plays the role of internal energy and we


have the relation, S = M2 /2. In particular, the temperature of the
black hole is:
1 dS 1
= =⇒ T =
T dM M
We may then write S = 1/2T 2 . The amount of heat required for
the process is therefore2 , 2
The negative sign is counter-
intuitive. This is related to the fact
Z ∞
1 1 that Schwarzschild Black Holes have
Z Z
dQ = TdS = − dT = − a negative specific heat. The black
T T2 T
hole emits heat while increasing it’s
temperature.
Therefore, a = b = −1 and correspondingly a − b = 0 .

3. Consider a quantum system described by a time-independent


Hamiltonian Ĥ. Let |ψ1 i , |ψ2 i be two orthonormal eigenvectors of
Ĥ with eigenvalues E1 and E2 respectively. Suppose at t = 0, the
system is prepared in the state,

1
|φ(0)i = √ (|ψ1 i + |ψ2 i)
2

and allowed to evolve with the Hamiltonian Ĥ. Let |φ(t)i be the
h
state at time t and let T = . Compute hφ( T )|φ(0)i.
2( E1 − E2 )

Solution. This is a very straightforward problem. The evolution of


an eigenfunction of a time-independent Hamiltonian is governed
by:  
iEn t
|ψn (t)i = exp − |ψn (0)i

where |ψn i is the eigenfunction and En is the corresponding eigen-
value.
Using this and the fact that3 , 3
Note that the quantity in the numera-
tor is h = 2πh̄ and not h̄.
h
T=
2( E1 − E2 )
we get:
hφ( T )|φ(0)i = 0

4. An ideal gas (non-interacting) of particles of mass m at tempera-


ture T is subjected to a constant external gravitational field with
potential energy:
U ( x ) = mgz
a
where 0 ≤ z < ∞. The average energy of each particle is: k B T.
b
What is a + b?
hints and answers stems 2021 physics category c qualifier round 3

Solution. Since the particles are non-interacting, it suffices to


compute the average energy of a single particle. The average total
energy of a single particle,

h E i = h T i + hU i

is the sum of the average kinetic and potential energies. The aver-
age kinetic energy for an ideal gas is:
3
hT i = kB T
2
The average potetial energy is computed via4 , 4
We use the canonical ensemble since
R the system is at a constant temperature
mgz exp(− βmgz) dz T.
hU i = R
exp(− βmgz) dz

where β = 1/k B T.
The denominator is trivial to integrate,
Z ∞
1
exp(− βmgz) dz =
0 βmg

To evaluate the numerator we set x = βmgz and so,


Z ∞ Z ∞
1
mgz exp(− βmgz) dz = 2
x exp(− x ) dx
0 β mg 0

Following the hint in the question, the integral evaluates to Γ(2) =


1. Finally we get:
1 5
hU i = = k B T =⇒ h Ei = k B T
β 2

Hence, a = 5, b = 2 and a + b = 7 .

5. Let ~a and ~b be two orthogonal vectors in the usualhthree dimen-


i
~
sional space with magnitude π each. Compute, Tr ei~σ·~a ei~σ·b ,
where ~σ = (σ1 , σ2 , σ3 ) are the standard Pauli sigma matrices for
spin-1/2 and Tr[ A] is the trace of a matrix A.

Solution. There are several ways to solve this problem but the
simplest is to use the identity,
σ · a
exp(iσ · a) = cos( a) + i sin( a)
a
√ q
where a = a · a = a21 + a22 + a23 .
The identity is easily derived by expanding the exponential.
Hence,
Tr [exp(iσ · a) exp(iσ · b)] = 2
hints and answers stems 2021 physics category c qualifier round 4

6. Consider a point charge q placed at a distance r above an infinite


conducting plane. At time t = 0 we start moving the charge with
a speed v parallel to the plane. Say there is some time lag (because
information cannot travel instantaneously) and the image charge
starts moving at t = τ. Suppose v is small so that the process is Based on a problem in the Boston Area
quasi-static and Coulomb’s law is still valid. Also suppose that τ is Undergraduate Physics Competition
(BAUPC).
small. The leading contribution to the force in the direction of v is:
kq2 τ
F= v
Nr3

What is the value of N?

Solution. The distance between the charge and it’s image charge
is,
d2 = (2r )2 + (vτ )2 2r
d
The force in the direction of v is therefore,
kq2 vτ
F=
d2 d
In a first order approximation, we may write d ≈ 2r. Hence, Figure 1: The separation between the
N=8. charge and it’s image is d.

7. A drunkard performs a random walk on an infinite plane after a


party. Due to increasing fatigue, each step taken is 0.99 times the
previous one. If the initial step was 1 unit, the RMS distance from
the starting point after N steps is L N . Compute,

lim L N .
N →∞

(Express your answer to one decimal place)

Solution. We will take the Argand plane and use complex num-
bers instead of R2 .
The position of the drunkard in the argand plane after n steps is:

zn+1 = exp(iθ0 ) + 0.99 exp(iθ1 ) + · · · + (0.99)n exp(iθn )

where θ0 , θ1 , . . . , θn are the directions of the first, second, . . . , nth


step respectively.
Figure 2: A typical path of the drunk-
We write this as, ard (figure not to scale)
n
z n +1 = ∑ (0.99)k exp(iθk )
k =0

Now,
n
|zn+1 |2 = z∗n+1 zn+1 = ∑ (0.99)k+ j exp(i(θk − θ j ))
k,j=0
hints and answers stems 2021 physics category c qualifier round 5

Say Ln is the RMS value of the position of the drunkard, then:


n n
L2n = h|zn+1 |2 i = ∑ (0.99)k+ j hexp(i(θk − θ j ))i = ∑ (0.99)2k
k,j=0 k =0

where we used the fact that, hexp(i (θk − θ j ))i = δjk when θk and θ j
are random.
If we take the limit n → ∞, we get:
s
1
lim Ln = ≈ 7.1
n→∞ 1 − (0.99)2

rounded off to one decimal place.

You might also like